Help I need the right answer

Help I Need The Right Answer

Answers

Answer 1

Answer:

Daisy ran the greatest distance while Carlos ran the shortest

Step-by-step explanation:

In fraction the bigger the number looks the smaller it actually is because it fraction you splitting things into smaller halfs not increasing them

Answer 2
This will be easy if we write them in decimal
1/3 = 0.333
3/5 = 0.6
7/10= 0.7
1/2=0.5
The largest distance is 0.7 who is Carlos
And the lowest one is 0.333 who is Angie

Related Questions

Which statement explains how you could use coordinate geometry to prove that quadrilateral ABCD is a rectangle ​

Answers

Answer:

prove that opposite sides are congruent and parallel

Step-by-step explanation:

i took the testtttttt

The statement which explains is Prove that opposite sides are congruent and that the slopes of consecutive sides are opposite reciprocals, the correct option is D.

We are given that;

The quadrilateral ABCD

Now,

A rectangle is a quadrilateral with four right angles, which means that the slopes of consecutive sides are negative inverses of each other.

For example, if the slope of AB is m, then the slope of BC is -1/m.

Also, a rectangle has opposite sides that are equal in length,

which means that the distance between A and B is the same as the distance between C and D, and the distance between B and C is the same as the distance between A and D.

Therefore, by quadrilateral the answer will be prove that opposite sides are congruent and that the slopes of consecutive sides are opposite reciprocals.

To learn more about quadrilateral visit;

https://brainly.com/question/29934440?referrer=searchResults

#SPJ4

the difference of 17 and 5 times a number

Answers

Answer:

I need more context

Step-by-step explanation:

The angle measurements in the diagram are represented by the following expressions

Answers

Answer:

∠A is equal to 73°

Step-by-step explanation:

Because the two lines that move roughly bottom/left to top/right are marked as being parallel, we can say that ∠A + ∠B = 180°

With that we can take the two equations at the top, and add them together for a sum of 180°:

6x - 35° + 3x + 53° = 180°

9x + 18° = 180°

x + 2° = 20°

x = 18°

Now that we have x, we can solve for ∠A:

∠A = 6x - 35°

∠A = 6(18°) - 35°

∠A = 108° - 35°

∠A = 73°

We can test our answer by solving for ∠B.  We know that ∠A and ∠B add up to 180°, so ∠B = 180° - ∠A = (180 - 73)° = 107°.  Now let's solve for be given x and the original equation, and see if we have the right numbers:

∠B = 3x + 53°

∠B = 3(18°) + 53°

∠B = 54° + 53°

∠B = 107°

Which matches up, showing us that we have the correct answer.

Can someone help me please

Answers

Answer:

(3x+1)(x+9)

Step-by-step explanation:

Answer:

(3x+1)(x+9) is the answer

Which of the following is a true polynomial identity?
a) a^6+b^6 = (a^2 + b^2)(a^4 + a^2b^2 + b^4)
b) a^6 - b^6 = (a^2+b^2)(a²-ab+b²)(a²+ab + b²)
c) a^6 - b^6 = (a³-b³)(a-ab+b)(a²+ab+b²)
d) a^6 - b^6 = (a²-b²)(a²-ab+b^2)(a^2+ab+b^2)​

Answers

c or d i think is the correct answer

cuánto es 567 + 678 es igual

Answers

Answer: 1245 :)

Step-by-step explanation:

Answer:

la respuesta es 1245

567+678=1245

2/7 letts gooo keep it goo yall doing great

Answers

Answer:

all of them

Step-by-step explanation:

I'm pretty surep

SOMEONE PLEASE EXPLAIN THIS TO ME

Answers

17.5 = x

Step-by-step explanation:

1st you add 85 and 55 since those to angles are congruent (means similar) to 8x

2nd you get 140 after adding then

3rd since the angles are congruent you just divide the 140 by 8 to get the value of x

14b-12b-3? plss helpp

Answers

If it’s to simplify its 2b-3

5. What are the values of x and y? Be sure to show your work.
helpppp plz hurry

Answers

Answer:

x=35 y=72.5

Step-by-step explanation:

x is equal to 35 because they are vertical angles. y is equal to 72.5 because if you use the equation y + y + 35 = 180 and subtract 35 then divide 145 by 2, you would get 72.5

using the commutative property, what would be the equivalent expression for 5+6= ?

Answers

5+6=? It would be mostly 11

Which triangle makes this statement true?

Answers

Answer:

Step-by-step explanation:

pls answer this if you do I mark brainlest​

Answers

Answer:

268

Step-by-step explanation:

2×(10×8 + 10×3 + 8×3) = 268 yards

Answer:

[tex]268yd^2[/tex]

Step-by-step explanation:

The Surface Area formula for a rectangular prism is:

[tex]2(wl+hl+hw)[/tex]

where w is the width, h is the height, and l is the length.

Plug each value into the formula and solve:

[tex]2(8*10+3*10+3*8)=268yd^2[/tex]

If this helped, a brainliest answer would be greatly appreciated!

Which polynomial represents the sum below? (4x5 + 6x3 + 3) + (3x3 + x + 9)​

Answers

Answer:

i think

 4x^5+9x^3+x+12

Step-by-step explanation:

A sample from a population with μ = 40 and σ = 8 has a mean of M = 36. If the sample mean corresponds to a z = –1.00, then how many scores are in the sample?

Answers

Answer:

There are 4 scores in the sample.

Step-by-step explanation:

To solve this question, we need to understand the normal probability distribution and the central limit theorem.

Normal Probability Distribution:

Problems of normal distributions can be solved using the z-score formula.

In a set with mean [tex]\mu[/tex] and standard deviation [tex]\sigma[/tex], the zscore of a measure X is given by:

[tex]Z = \frac{X - \mu}{\sigma}[/tex]

The Z-score measures how many standard deviations the measure is from the mean. After finding the Z-score, we look at the z-score table and find the p-value associated with this z-score. This p-value is the probability that the value of the measure is smaller than X, that is, the percentile of X. Subtracting 1 by the pvalue, we get the probability that the value of the measure is greater than X.

Central Limit Theorem

The Central Limit Theorem estabilishes that, for a normally distributed random variable X, with mean [tex]\mu[/tex] and standard deviation [tex]\sigma[/tex], the sampling distribution of the sample means with size n can be approximated to a normal distribution with mean [tex]\mu[/tex] and standard deviation [tex]s = \frac{\sigma}{\sqrt{n}}[/tex].

For a skewed variable, the Central Limit Theorem can also be applied, as long as n is at least 30.

In this question, we have that:

[tex]\mu = 40, \sigma = 8, X = 36, Z = -1, s = \frac{8}{\sqrt{n}}[/tex]

We want to find n. So

[tex]Z = \frac{X - \mu}{\sigma}[/tex]

By the Central Limit Theorem

[tex]Z = \frac{X - \mu}{s}[/tex]

[tex]-1 = \frac{36 - 40}{\frac{8}{\sqrt{n}}}[/tex]

[tex]-4\sqrt{n} = -8[/tex]

[tex]4\sqrt{n} = 8[/tex]

Simplifying by 4

[tex]\sqrt{n} = 2[/tex]

[tex](\sqrt{n})^2 = 2^2[/tex]

[tex]n = 4[/tex]

There are 4 scores in the sample.

Joel races bikes. In his city, there are 4 4 lengths of race tracks. The chart shows the race tracks and each track's length in miles. Race Track Length in Miles A 1 4 4 1 ​ B 3 2 2 3 ​ C 1 3 4 1 4 3 ​ D 4 4 4 4 ​ Joel races 3 3 laps in the same lane. The total distance he races is less than 3 3 miles. Which track does Joel race on?

Answers

So if Joel raced 3 laps , and the total distance is less then 3 miles then it has to be Track C.

The sum of 7 and twice a number is 4. Write an equation.

Answers

7 + 2x = 4 (I’m under the word limit just ignore this)

marco set aside 50 minutes

Answers

Answer:

So what's the Question Because to me it looks like something random

6.3.12
The coordinates of point T are (0,5). The midpoint of ST is (2,-2). Find the coordinates of point S.
The other endpoint is
(Type an ordered pair.)

Answers

Answer:

[tex]S = (4,-9)[/tex]

Step-by-step explanation:

Given

[tex]T = (0,5)[/tex]

[tex]Midpoint = (2,-2)[/tex]

Required

Find S

Midpoint is calculated as:

[tex]Midpoint = 0.5(x_1 + x_2,y_1+y_2)[/tex]

So, we have:

[tex](2,-2)= 0.5(0+ x_2,5+y_2)[/tex]

Where x2 and y2 are the coordinates of S

Multiply through by 2

[tex]2 * (2,-2)= 2 * 0.5(0+ x_2,5+y_2)[/tex]

[tex](4,-4)= (0+ x_2,5+y_2)[/tex]

[tex](4,-4)= (x_2,5+y_2)[/tex]

By comparison:

[tex]x_2 = 4[/tex]

[tex]5 + y_2 = -4[/tex]

[tex]y_2 = -4-5[/tex]

[tex]y_2 = -9[/tex]

So:

[tex]S = (4,-9)[/tex]

Corrie breathed 1 × 10^7 times in a year. She blinked 8 × 10^6 times that year. How many more times did Corrie breathe in a year than she blinked in that year? Write your answer in standard notation

Answers

Snskskskjssksj28828384838188:$2&82:&/&:&

Step-by-step explanation:

1 × 10 ^7 - 8 × 10 ^ 6

2 × 10 ^ 6 which is also equivalent to 2,000,000

5(t+4) please heelllpppp

Answers

Answer:

5t+20

Step-by-step explanation:

Suppose we want to choose 5 objects, without replacement, from 9 distinct objects. (a) How many ways can this be done, if the order of the choices is not taken into consideration? (b) How many ways can this be done, if the order of the choices is taken into consideration?

Answers

The number of ways if the order of the choices is not taken into consideration and taken into consideration will be 126 and 15,120, respectively.

What are permutation and combination?

A permutation is an act of correctly arranging things or pieces. Combinations are a method of taking stuff or pieces from an assortment of items or sets in which the order of the items is irrelevant.

Suppose we want to choose 5 objects, without replacement, from 9 distinct objects.

Then the number of ways that this can be done, if the order of the choices is not taken into consideration will be given as,

⁹C₅ = 9! / [(9 - 5)! x 5!]

⁹C₅ = (9 x 8 x 7 x 6 x 5!) / (4 x 3 x 2 x 1 x 5!)

⁹C₅ = 126

Then the number of ways this can be done, if the order of the choices is taken into consideration will be given as,

⁹P₅ = 9! / [(9 - 5)!

⁹P₅ = (9 x 8 x 7 x 6 x 5 x 4!) / 4!

⁹P₅ = 15,120

The number of ways if the order of the choices is not taken into consideration and taken into consideration will be 126 and 15,120, respectively.

More about the permutation and the combination link is given below.

https://brainly.com/question/11732255

#SPJ2

The entire graph of the function f is shown in the figure below.
Write the domain and range of f as intervals or unions of intervals.

Answers

you need to put a picture of the graph

difference quotient for f (x) = 2x + 3 /5x

Answers

Answer:

1 and -0.6

Step-by-step explanation:

2. In triangle ABC, SA is a right angle, and m B = 45°
(1 point)
C
16 ft
A
B.
What is the length of BC? If your answer is not an integer, leave it in simplest radical form.
O 16 ft
O 16.2 ft
O 16.5 ft
O 32 ft

Answers

Answer:

[tex]16\sqrt{2}[/tex]

Step-by-step explanation:

The given triangle shown is a 45-45-90. We can use this picture attached to figure out BC.

n in this case is 16ft

Based on the picture, we can get BC's length, which is n[tex]\sqrt{2}[/tex]

BC = [tex]16\sqrt{2}[/tex]

A dilation produces a smaller figure. Which is a possible scale factor?
0 -2
O 1/3
O 1
O4

Answers

Answer:

1/3

Step-by-step explanation:

tfewfgeeffeefefefwefwewwefewewfefwfew

Answers

Answer:

I understand your pain

Step-by-step explanation: sadge

25 + 12x (24 + 3) - 15 21.3

Answers

Use this. It’s really good

Answer:

324x-1496.3

Step-by-step explanation:

What is the range of f(x) = (three-fourths) Superscript x – 4?

Answers

{y | y > –4}



Left-brace y vertical line y greater-than three-fourths right-brace



{y | y < –4}



Left-brace y vertical line y less-than three-fourths right-brace

Answer:

What is the range of f(x) = (three-fourths) Superscript x – 4?

* {y | y > –4}

** Left-brace y vertical line y greater-than three-fourths right-brace

*** {y | y < –4} <<<correct

**** Left-brace y vertical line y less-than three-fourths right-brace

Step-by-step explanation:

Edge 2021

Help please !!!! This is really hard

Answers

Answer:

Option -c  :5^ -12  .  2^ 40

Step-by-step explanation:

(  5^6 . 2^8    )( 5^-2  . 2^5 )

(  5^6 . 5^-2  )   ( 2^8  . 2^5 )

5^ -12  .  2^ 40

Option -c :  5^ -12  .  2^ 40

I hope im right!!  

Other Questions
Which equation matches this graph?y=(1) +y = 4 +3-3 -2 -10y=-()* +3y = -4" + 3 PLEASE HELP ME I NEED HELP RIGHT AWAY!!!A box contains fiction and nonfiction books. There are 22 fiction books in the box, which is 55% of the total number of books in the box. What is the total number of books in the box?thank you for helping Why is earth considered an open system and a closed system? (Site 1)O What were three effects on f westward expansion Dan has 3 cups of milk. If he used of the milk for his breakfast, how much milk does Dan have left? Why did Toussaint Louverture lead the Haitian Revolution? who are the people involved in making motion information? Can you please solve this Do you believe that animals should be used tohelp advance healthcare for humans? elimination methodx-5=-y3x-10=2y how did marcus garveys approach to civil rights differ from that of the NAACP you are playing rock , paper , scissors with your friend . what is the probability that your will throw something other than rock ? A. 1/2 B. 2/3 C. 1 D. 1/3 Pls answer this for me Ensemble Co.Unadjusted Trial BalanceFor the Year Ending December 31, 2018Debit BalancesCredit BalancesCash42,900Accounts Receivable123,500Prepaid Insurance27,000Equipment300,000Accounts Payable52,000Salaries Payable4,800Common Stock40,000Retained Earnings137,200Dividends5,000Service Revenue1,216,000Salary Expense660,000Advertising Expense275,000Miscellaneous Expense16,6001,801,5001,801,500How does grading work?Ensemble Co.UNADJUSTED TRIAL BALANCEACCOUNT TITLEDEBITCREDIT1 Cash2 Accounts Receivable3 Prepaid insurance4 Equipment5 Accounts Payable6 Salaries Payable7 Common Stock8 Retained Earnings9 Dividends10 Service Revenue11 Salary Expense12 Advertising Expense13 Miscellaneous Expense14 Totals Find the volume of the solid obtained by rotating the region bounded by the given curves about the specified line. Sketch the region, the solid, and a typical disk or washer. y2=x,x=2yy 2 =x,x=2y; about the y-axis I need to know how to find the measure of angle a. Why do Jack and Jill free the thief?because he can protect thembecause he is not an honest thiefbecause he knows the way outbecause he knows how to save their mother What is the best English translation for "Visum est?" what is job description A 50 kg mass is sitting on a frictionless surface. An unknown constant force called force A pushes the mass for 2 seconds until the mass reaches a velocity of 3 m/s. If the 50 kg mass is now pushed by an unknown force B and reaches the velocity of 3 m/s in 4 seconds, compare the impulse delivered to the mass when acted upon by force A with the impulse delivered to the mass when acted on by force B? *A) The impulse delivered to the mass when acted upon by force A is greaterB) The impulse delivered to the mass when acted upon by force B is greaterC) The impulse is the same in each caseD) We need to know the value of force A and force B in order to determine this